Chapter 15

Practice Exam 1: Answers and Explanations

If this is the second practice exam you’ve taken, we hope you’ve improved your scores and are feeling more confident about taking the Praxis Core. Use the answers and explanations in this chapter to see how well you performed and to understand where you might have gone wrong on the answers you missed. Remember, the practice exam can help you determine where you need to focus your studies in preparation for the real Praxis Core. If you want to score your test quickly, flip to the end of the chapter, where the “Answer Key” gives only the letters of the correct answers.

tip If you want even more preparation for taking the Praxis, head to Dummies.com. There you can take more practice tests, review the answers and explanations, and get a personalized summary of your performance.

Part 1: Reading

  1. B. The modern use of the term sports focuses on traditional views solely focused on physical competition, while the author proposes that the study of sport can be a gauge to measure other aspects of particular cultures and time periods.

    The author explains that the terms are different, in that sports is a traditional term referring to our current understanding of the topic with teams, players, and so on. Sport, however, is a term that encompasses viewing sport over time and can be used to better understand a certain civilization and time period in history.

    The correct answer is not Choice (A) because Sports, as a term, can also refer to historical actions or events.

    The correct answer is not Choice (C). The point of the passage is that the author does not want the terms used interchangeably. In this context, he is stating that the terms mean different things.

    The correct answer is not Choice (D) because it’s an oversimplification. The author is suggesting that sport might include other elements, but doesn’t necessarily state what those might be. In this paragraph, both terms still refer to some type of physical feat.

    The correct answer is not Choice (E). The author doesn’t suggest that one term is preferable or that one should be considered superlative in some way.

  2. A. In order to satisfy the public, Homer appealed with tales of epic heroic deeds.

    The prompt asks for a supposed purpose for the writer Homer. The passage states that Homer’s patrons “mandated that he applaud” heroes from a greater age of men. So, his purpose in writing was to appease his patrons.

    The correct answer is not Choice (B) because the prompt, and this part of the passage, isn’t concerned with solving the “Homeric question.”

    The correct answer is not Choice (C) because this choice attempts to solve the “Homeric question,” which isn’t the purpose of the prompt or the purpose of the passage. So, his purpose in writing was to appease his audience.

    The correct answer is not Choice (D) or (E). While both are true statements, they do not address the prompt. The prompt asks for the purpose of the writer Homer in writing such tales.

  3. D. thinking that involves an emphasis on an appreciation of nature and the common man.

    While the term “romanticism” can mean all of these things, here, “romanticism” is referring specifically to the thinking of seeing events with an appreciation of nature and the common man. Evidence is seen using the terms “bucolic” for natural settings and for “the people who gave us democracy.”

    The correct answer is not Choice (A). While the examples given are ones of a positive note, the tone stops short of idyllic.

    The correct answer is not Choice (B). Not all examples in the passage describe a rural life.

    The correct answer is not Choice (C). While the term “romanticism” is often used this way, the passage does not suggest that the view is suddenly against previous views. Rather, the romantic view is the one that is traditional and long-held.

    The correct answer is not Choice (E). This answer would best fit the word “haze,” but not the term “romanticism.”

  4. D. One myth of the cultural sports spectacle is that Greek sports have been wiser and better, while Roman sports have debased man’s nature.

    The author states the myth supposing that Greeks are somehow more positive than Romans without giving further examples to prove or disprove this.

    The correct answer is not Choice (A). The author does not suggest that Romans are more deserving of a positive viewpoint.

    The correct answer is not Choice (B). The author does not allude to any evidence that would support either side.

    The correct answer is not Choice (C). While man has historically cast one culture in a negative light, more evidence would be needed to prove this conclusion as true.

    The correct answer is not Choice (E). No evidence is given to prove that the myth against Romans is untrue.

  5. A. Hollywood has perpetrated a false image of gladiators that isn’t supported by historical evidence.

    The phrase “despite Hollywood” leads the reader to understand that this depiction of gladiators as being forced to perform to death as slaves is wrong. Rather, history shows that gladiators had many roles in society.

    The correct answer is not Choice (B). Gladiators weren’t necessarily Spanish slaves. The word “gladiator” is from a Spanish root.

    The correct answer is not Choice (C). The passage gives no details about general assumptions. The point of the text is that Hollywood has led to misunderstandings about history.

    The correct answer is not Choice (D). According to the passage, gladiators have been both in history. Hollywood, though, has only portrayed them in one way.

    The correct answer is not Choice (E). Hollywood actually supported the opposite — that gladiators were helpless slaves forced to perform.

  6. C. hydro

    Hydro has seen the steadiest use of energy consumption, which has varied little between the years 1800 and 2008.

    The correct answers are not Choices (A), (B), (D), or (E) as all of these have seen sharp increases in certain years of consumption.

  7. B. Where future generations are born determines their per capita consumption rates.

    The author is suggesting that, when forecasting energy demand, it matters greatly where those future generations are born as that determines to a large extent their per capita consumption rates.

    The correct answer is not (A). The author gives factors for determining future energy demands.

    The correct answer is not Choice (C) or (D). While Americans do consume more now, the population growth will likely not keep up with developing nations, which makes this a false forecast. However, should that change, then the developing nations will not have the consumption rates in greater numbers. The rate varies due to the population. More people = more energy consumption in the future.

    The correct answer is not Choice (E). The author makes no direct causal effect between future consumption rates and standard of living.

  8. E. Although one might expect sexism to negatively affect women and girls, men and boys can also be targets of this kind of discrimination.

    While many expect to hear about gender discrimination against women and girls, men and boys also suffer and are victims.

    The correct answer is not Choice (A). While this is an assertion, this is not necessarily a conclusion from the text.

    The correct answer is not Choice (B) or (D) because no examples are given as to how sexism could be positive.

    The correct answer is not Choice (C). While this is also an assertion, it is not the best answer. The point the author is trying to make is that for men to be victims of this type of discrimination is unusual, because it generally is believed to only affect females.

  9. A. In short, these and other studies suggest that boys tend to suffer more than girls as a result of divorce and of living with a single parent.

    One can conclude that the author’s final statement would be suggesting that the studies show similar information in regards to the suffering of boys postdivorce.

    The correct answer is not Choice (B). After reading the snippets from the studies, the readers should not be surprised by such a conclusion.

    The correct answer is not Choice (C) because the reader has no cause to question what the studies might have been lacking.

    The correct answer is not Choice (D). The parents of children who suffer from the consequences of divorce aren’t likely to feel better by a study that shows they are the cause of a child suffering because of these very actions.

    The correct answer is not Choice (E). The author presents enough cohesive information and details to make a logical prediction about the continuation of the text.

  10. C. emphasize the danger of differentiating what is considered worthy, from a scholar’s perspective, from an attitude of popular perspective.

    The example given shows the danger in teaching only some of a subject or topic. If some knowledge is considered “good” and some “bad,” then this knowledge will be acted upon and can be swayed by popular opinion.

    The correct answer is not Choice (A). Faith and religion are used for the same purpose in this passage. These terms are not representing different concepts.

    The correct answer is not Choice (B). The example given is one of using faith as knowledge, but doesn’t explain why this happened so frequently in our past, only that it did often happen.

    The correct answer is not Choice (D). The point is that knowledge shouldn’t be considered on the basis of “good” or “bad.” Knowledge should be shared in an unbiased way for its own sake.

    The correct answer is not Choice (E). Here, hindsight clearly shows what was “bad” to teach since it resulted in deaths by unfounded witch hunts.

  11. E. A generalization is made followed by a specific example.

    The author makes a claim and gives general examples. Then, she gives a specific example to more clearly illustrate the original point.

    The correct answer is not Choice (A). No metaphor is given, although figurative language is used to liven up the topic.

    The correct answer is not Choice (B). The author provides no objections to her claim.

    The correct answer is not Choice (C). There are no comparisons or contrasts.

    The correct answer is not Choice (D). The claim isn’t “suspected.” The author is quite clear on her viewpoints of the topic.

  12. C. Critical readings and writings by this group have not been treated with serious academic respect.

    The author says such thinkers have been “scapegoats” and their thinking was little more than “pleasant fancy.” Clearly, the author is suggesting no serious academic respect has been given to these views.

    The correct answer is not Choice (A). The author is suggesting the opposite of this statement.

    The correct answer is not Choice (B) because the author provides no “merit” for such treatment.

    The correct answer is not Choice (D) because the author suggests no one has really taken the ideas seriously.

    The correct answer is not Choice (E). The ideas have been dismissed, but a serious female academic Shakespeare expert was noted. Alas, her work was treated scornfully.

  13. D. strengthen.

    Marriages during this time were often done for advantageous political purposes, meaning that such a marriage would strengthen relationships between the allegiances.

    The correct answer is not Choice (A). While this is a synonym of “cement,” it is not the best answer choice and does not quite fit the situation described.

    The correct answer is not Choice (B) or Choice (C). Both words could be considered antonyms or near antonyms.

    The correct answer is not Choice (E). Such a marriage might bring new life to political alliances that were stale, for example. However, the examples listed show the marriages provided a strength for these men and is the reason they kept several wives.

  14. D. lend credence to the idea that Caesar cared more about Cleopatra’s power than the idea of love.

    Because Caesar was considered a “womanizer,” he likely saw Cleopatra more as an advance to his own political goals and less as a traditional wife.

    The correct answer is not Choice (A). Because he was understood to be a womanizer, this would be his normal moral code of character.

    The correct answer is not Choice (B). No additional details in the passage support this idea.

    The correct answer is not Choice (C). No other information indicates that accepted views of Caesar are incorrect.

    The correct answer is not Choice (E). The passage states that there are fewer sources of descriptions of this marriage and more for Cleopatra’s well-known love match, that of Antony.

  15. E. A controversial claim is presented, and then evidence and reasons are given to support one side of the controversy.

    Trade deficits and energy usage is a controversial subject and one that can be argued with positive and negative points. Here, the author makes a direct claim and call to action and then gives reasons as to why this call to action should be followed.

    The correct answer is not Choice (A). No unresolved question is stated.

    The correct answer is not Choice (B) or Choice (D). A “movement” is not the best way to describe America’s energy independence on other nations.

    The correct answer is not Choice (C). Since the issue is ongoing, this isn’t the best answer choice. Also, there is no one pivotal moment described.

  16. D. a desire to sell works to a specific audience.

    The author states that any editor begins by thinking about what the audience wants. This segues into a discussion about book markets.

    The correct answer is not Choice (A). While annotation has expanded in books, the appeal is not necessarily for a broader range. Rather, the appeal is for a specific audience—the undergraduate markets.

    The correct answer is not Choice (B). The passage states this is a consideration when designing books, not that it has led to more annotations.

    The correct answer is not Choice (C). Rather, more annotation has led to a cultivation of critical methodology and literary relevance.

    The correct answer is not Choice (E). The passage gives no specific detail concerning how publishing has changed, only how annotation has changed.

  17. B. explain that evidence bears out the understood assumption that Ptolemy was a weak ruler

    The author first states that Ptolemy XII is understood to be a weak ruler because he was a prolific builder. The gateway at the Ptah temple bears this out because “it is small and would not have cost a great deal of money.”

    The correct answer is not Choice (A). According to the author the prolific building projects mean the exact opposite.

    The correct answer is not Choice (C). The point of the passage is that the author does not want the terms used interchangeably. In this context, he is stating the terms mean different things.

    The correct answer is not Choice (D) or Choice (E) because the information presented about the gateway supports the author’s first statement rather than refutes it.

  18. C. sensationally touted.

    Here, the word means that the relationship with China has been sensationally touted.

    The correct answer is not Choice (A), Choice (D), or Choice (E). While the word does have these multiple meanings, these nuances don’t fit the context of the sentence.

    The correct answer is not Choice (B) because the topic, the conversation concerning a trade deficit, is a substantial one. Again, the word nuance doesn’t quite fit here.

  19. D. a decrease in traffic-related fatalities.

    The passage states a decrease in the number of people killed in traffic accidents was the silver living in the gas shortage, suggesting that the decrease in deaths could be attributed to this.

    The correct answer is not Choice (A). The author doesn’t make this claim necessarily. Choice (D) is the most obvious answer.

    The correct answer is not Choice (B). The author doesn’t suggest the lowered speed limits were a result of the gas shortage. A reader might assume this, but the direct correlation the author draws is the link between less gas and fewer deaths. Again, just not the best choice.

    The correct answer is not Choice (C). No real information is given about the overall energy shortage, so there is no correlation between the two.

    The correct answer is not Choice (E). Naturally, these two are linked. To make a direct correlation between them would have been a waste of time for an author.

  20. C. It was worn during his first fireside chat.

    The only information given is that the sweater was worn during Carter’s first fireside chat.

    While all the other answer choices could be true as symbolic statements, the passage doesn’t suggest any of these nor provide evidence for them. So, Choices (A), (B), (D), and (E) are incorrect.

  21. A. When assessing how much it costs to fight terrorism in the Persian Gulf, consideration has to be made for multifaceted effects.

    The passage states that, while there have been many military fatalities, there have also been numerous other traumatic effects, such as the loss of life to journalists and innocent Iraqis, as well as the suffering of the families of those involved and a huge monetary cost.

    The correct answer is not Choice (B). The author makes claims that are reasonable and don’t need future evidence to support them.

    The correct answer is not Choice (C). While a number is given for military deaths, other data is not available here for comparison.

    The correct answer is not Choice (D) or Choice (E). Both are opinion statements that are not supported by evidence provided in the text.

  22. C. pointing out that the film Olympia gave credence to the Berlin Olympics through the propagandistic use of symbols featuring honor and glory.

    The passage points out that the film Olympia is a propaganda film that features, and thus sanctions, the Berlin “Nazi” Olympics by comparing the sport to the “pure” Olympic Greek games.

    The correct answer is not Choice (A). There are no direct or indirect warnings. The author is simply pointing out how this film helped to advance Nazi propaganda techniques.

    The correct answer is not Choice (B). By stating the film is “propagandistic,” the author is stating it was not inadvertent. Rather, it was purposeful. Choice (D) is wrong for this same reason. The two are linked because the subject of the film is the Berlin Olympics.

    The correct answer is not Choice (E). The author does not purport the reader take any perspective, other than one of understanding how the film and Hitler are linked.

  23. B. while the film may have been made for nefarious reasons, Olympia has nevertheless changed cinematic history and inspired future sports films.

    Although the film was one of propaganda, it still changed the way that future sports films and documentaries would be made because of its careful attention to detail and focus on physical glory and honor. The author explains that the film should be viewed not for the propaganda, but for the way it changed how sports films were made and produced.

    The correct answer is not Choice (A). The passage does not state that Hitler designed the film. The film was the work of cinematographer Leni Riefenstahl.

    The correct answer is not Choice (C). The author is not stating or suggesting ideas on how sports films should be made.

    The correct answer is not Choice (D). The film is mentioned in connection with the Berlin Olympics because it is about the Berlin Olympics.

    The correct answer is not Choice (E). The passage does not state where the film was unveiled.

  24. C. explaining that sport permeates modern society and, while that is primarily viewed in a positive light, there are also many negative effects.

    The passage explains that sport is in every aspect of our culture and, while sport is generally viewed as a good thing, there are also some negative connotations. The author gives both positives (sports played at the YMCA and by schoolchildren) and negatives (the violence of sports and sport as a money-making business).

    The correct answer is not Choice (A). Rather, more time is spent with negative examples, than with positive examples.

    The correct answer is not Choice (B). The tone is not one of refutation of a previous argument. The author is simply making a point.

    The correct answer is not Choice (D). This is not simply a summary of facts. The author also has a point to make that might not occur to many or be viewed by the majority about the positive and negative aspects of sports in our culture.

    The correct answer is not Choice (E). The author does not ask or encourage the reader, in this section of the text, to make that determination — whether one aspect outweighs other considerations.

  25. B. While sport is seen in mostly a positive light, the negative side of the argument encompasses both violence and commercialization.

    In this passage, the author gives many examples as to how sports can be seen to have a negative effect regardless of the fact that many view it in a positive light.

    The correct answer is not Choice (A) because the author gives many examples as to how sport has affected culture.

    The correct answer is not Choice (C). While the author says sports traditions tend to still be viewed positively, he doesn’t propose this should be the case.

    The correct answer is not Choice (D) because the author doesn’t propose “culture” disagrees about sports. The author points out a potential disagreement about the influence of sports on modern society.

    The correct answer is not Choice (E) because the author does not call on the reader to make a judgment or decision.

  26. B. The energy consumption of countries can be directly related to the wealth and income of the people who reside there.

    The passage directly states the richest people in the world use more energy than those with less wealth. So, the conclusion is that energy consumption can be directly related to the wealth and income of the people who inhabit that country.

    The correct answer is not Choice (A). The information provided makes a clear case and provides no disagreement about the connection between energy consumption and economic development.

    The correct answer is not Choice (C). The variations are actually used to make the point and do not cloud the issue.

    The correct answer is not Choice (D). The author makes no distinctions regarding what would be “proper” or “improper” energy systems.

    The correct answer is not Choice (E). The word “extravagantly” suggests the author is placing blame for the use of energy resources, and this isn’t the case. The author is simply presenting the facts.

  27. A. often lack access to electricity and have to rely on biomass fuel options.

    The author states that people in developing nations suffer from energy poverty meaning they often don’t have access to electricity, which causes them to rely on other forms of energy, such as biomass fuel options.

    The correct answer is not Choice (B). This point seems to bring the reader to a Catch-22. Without the money, no development can begin, and with no development, no other energy options can be made available. However, the author’s main point is that a lack of access to electricity is the main reason for energy poverty, not just a lack of development options.

    The correct answer is not Choice (C). No information is provided about wealthier inhabitants of a country. The distinction is between the poorest and richest users the world over.

    The correct answer is not Choice (D). The author doesn’t suggest that the richest people have a monopoly on energy interests that isn’t sharable. Rather, they have an easier time accessing better energy options because of their wealth.

    The correct answer is not Choice (E). The author doesn’t suggest that energy poverty is an option chosen because it is in keeping with cultural norms. Rather, no other options are available to the poorest.

  28. E. a lack of access to energy.

    The author directly defines energy poverty as “the lack of access to energy services.”

    The correct answer is not Choice (A). No information is given about the interest of those who have no access to energy options.

    The correct answer is not Choice (B). While the energy-poor do also have a lack of resources, this isn’t the definition given for “energy poverty.”

    The correct answer is not Choice (C) or Choice (D). Naturally, the poor don’t have access to money, but this isn’t the definition given for “energy poverty.” Technically, a lack of money would simply be poverty.

  29. D. compare the huge discrepancies of energy consumption between the richest and poorest people in the world.

    The author states that the richest billion people in the world use 25 times more energy than the poorest 2.5 billion. These statistics are given to point out the huge discrepancies of energy consumption between these two groups.

    The correct answer is not Choice (A). Giving facts about the richest people wouldn’t help to explain why the poorest people have so few energy options.

    The correct answer is not Choice (B). No information is given about costs of energy development.

    The correct answer is not Choice (C) because the author rather supports this idea.

    The correct answer is not Choice (E) because the author supports the inference that those who were blessed being born in some areas of the world have better and richer lives than those who live in developing nations.

  30. D. How does not joining the military on a volunteer basis affect men?

    The passage explains the many ways that men are affected when they don’t voluntarily join the military. So, the question from Choice (D) could be answered by the passage. Effects given include shame, ostracism, and self-shame.

    The correct answer is not Choice (A). The passage does refer to contemporary western times, but gives no other indications this is the most affected time and place by this issue. Therefore, this question can’t be answered.

    The correct answer is not Choice (B). The author states it is hard for those in contemporary western culture to understand, but doesn’t give reasons as to why. So, this question can’t be answered.

    The correct answer is not Choice (C). This question can’t be answered because the passage doesn’t differentiate between different types of men or give reasons as to why some do choose to join and others don’t.

    The correct answer is not Choice (E). No reason is given as to why women are treated differently with regard to this issue, so this question can’t be answered.

  31. A. If men or boys choose not to join the military, there are clear effects from a societal and personal perspective.

    Many effects are given about a male reluctance to join the military, including ostracism and shame. This sentence best sums up the entire passage.

    The correct answer is not Choice (B) because only a small part of the passage is about how women are treated.

    The correct answer is not Choice (C). This does not summarize information in the entire passage and may only be an inference a reader can draw.

    The correct answer is not Choice (D) because this is a detail found in the text. It is not a summary of the entire passage.

    The correct answer is not Choice (E) because no details are given about this and the passage is not mostly about the lack of military enlistees.

  32. E. to illustrate key differences in the treatment of women and men in regard to voluntary military service

    The author shows how men and women are treated in the same situation — a reluctance to voluntarily join the military. The difference in the treatment received illustrates key differences between men and women and the repercussions for such a decision.

    The correct answer is not Choice (A) because women are treated much differently. According to the author, they aren’t shown any bias for making the same decision.

    The correct answer is not Choice (B) because the author does not suggest women are held in higher esteem, only that the repercussions for the decision are not as severe.

    The correct answer is not Choice (C). No reasons are given, other than gender, for why men are treated differently.

    The correct answer is not Choice (D) because the author doesn’t address this other than to say it exists.

  33. C. detailing ways in which men are more disadvantaged when it comes to a lack of respect of bodily privacy.

    The passage details the ways that men are actually more disadvantaged than women when it comes to a lack of respect for bodily privacy, such as differential treatment in prisons.

    The correct answer is not Choice (A). The author takes pains to point out the differences between gender treatment and doesn’t necessarily take pains to say each should be valued equally.

    The correct answer is not Choice (B) or Choice (D). The example in the prisons is given to make a point. The passage isn’t mainly concerned with how men or women are treated as prisoners.

    The correct answer is not Choice (E). The author is not suggesting that women should be treated worse or that they don’t enjoy their “better” treatment. The point is that men also suffer from this issue, although many may not realize it.

  34. B. females are more disadvantaged than males when it comes to bodily privacy in certain situations.

    In this example, the author specifically chooses a female clothing item to illustrate that females are more negatively influenced when it comes to issues of body privacy.

    The correct answer is not Choice (A) because the burqa is said to be worn by women in a way that shows a greater disadvantage for that gender in parts of the world where it is worn. An example that shows men are also negatively influenced comes later with the prison details.

    The correct answer is not Choice (C). While the author does give examples for both, the burqa is attributed to the female gender example.

    The correct answer is not Choice (D) because the author doesn’t suggest the issue is a cultural problem directed at only some segments of society. The issue is one of gender.

    The correct answer is not Choice (E) because the author does not make any statements concerning religion. Again, the issue is one of gender.

  35. E. Myths likely started as true stories or stories with some truth, but creativity has changed the tales over time so that no one really knows what is truth and what is imagination.

    While myths are enjoyable, many of the first stories were also based in truth, at least in part. Although some aspects are fantastical, other tales refer to events that actually happened in history.

    The correct answer is not Choice (A). Some notes are made suggesting that myths are likely based on some truths.

    The correct answer is not Choice (B). Myths include imaginative and creative aspects and can’t be simple retellings.

    The correct answer is not Choice (C). While it is hard to find elements of the truth, the examples given show that some elements of truth may be traced.

    The correct answer is not Choice (D). The author doesn’t give these two authors any more credibility than other unnamed authors.

  36. B. the stories make man look larger than life and the events take on a more heroic perspective.

    The author discusses how creative elements have been added that blend the divine and dangerous with human abilities and celebrate historic achievements of our past.

    The correct answer is not Choice (A). While myths are old and traditional forms of storytelling, the author gives many more reasons for why they have lasted so long.

    The correct answer is not Choice (C). Myths most likely started as tales around campfires, but are now shared in other situations.

    The correct answer is not Choice (D). The author infers that man’s achievements have been elevated, but also takes pains to point out elements that are obviously unrealistic. So, this is not the best answer choice.

    The correct answer is not Choice (E) because it is unclear whether these “heroes” were real, actual individuals.

  37. E. contained more imaginative aspects with less reliance on historical accuracy.

    The Epic of Gilgamesh contained divine aspects of gods and goddesses, while the other two focused on a historical event.

    The correct answer is not Choice (A). The Epic of Gilgamesh is likely older, as it is the oldest surviving myth, but no claim is made that this makes it a purer form of myth.

    The correct answer is not Choice (B). The Epic of Gilgamesh is less historically accurate as it contains divine, and therefore, imaginative aspects that are not real.

    The correct answer is not Choice (C). The author suggests all are examples of traditional myths even though they are different.

    The correct answer is not Choice (D). It is unclear whether early writers of myths wanted their tales to be remembered as truth or as stories.

  38. E. illustrating the difficulties faced when attempting to differentiate what is truth and what is fiction in ancient myths.

    The passage mostly discusses the difficulties modern people have when reading myths and determining what is true and what is made up.

    The correct answer is not Choice (A) because no new information or details are present. The author is relying on the understanding that modern man and ancient man have a commonality in that both suppose some aspect of those myths are true.

    The correct answer is not Choice (B). No assessment is detailed as being able to distinguish truth from fiction.

    The correct answer is not Choice (C) because the only literary classification listed as that of “myth.”

    The correct answer is not Choice (D) because no process is detailed nor is there information on who ultimately makes the determination that a story should or should not be a myth.

  39. C. stories that were written in a specific format of poetry.

    An epic poem is a long and serious narrative poem that features a hero, such as Odysseus.

    The correct answer is not Choice (A). Just because the poems are longer doesn’t mean they are more likely to be true.

    The correct answer is not Choice (B) or Choice (E). The author only lists the Homeric poems as examples, not the most impressive examples. The Epic of Gilgamesh is also listed and neither is suggested as being “better” than the other.

    The correct answer is not Choice (D). The author previously states this. This is not a reference or understanding of the term “epic poem.”

    The correct answer is not Choice (E). The author doesn’t provide any examples of works that were more outstanding, nor does he provide an opinion on the literary quality of the tales of Gilgamesh.

  40. A. Biomass has remained at a relatively stable usage throughout the years.

    Biomass fuel usage has remained relatively stable from 1800 to 2008.

    The correct answer is not Choice (B). The usage of biomass hasn’t changed greatly and appears to have no connections to other fuel forms used.

    The correct answer is not Choice (C). The introduction or invention of other fuel forms does not seem to have affected biomass fuel use.

  41. A and C. The introduction of alternative fuel sources has increased dramatically during recent times.

    Except for biomass, usage of all fuel types has increased more during the last 40–50 years than in the previous century.

    Both statements are simultaneously true. Using information from the chart, the reader can see that alternative fuel sources have increased dramatically during recent times. While the chart traces back more than 200 years, usage of all fuel types except biomass is seen as increasing since approximately 1970. The previous century, the 1800s, saw little or no increase in other forms of energy usage.

    The correct answer is not Choice (B). Because biomass fuels have remained steady, there has been no negative impact on other usage types, at least according to this chart.

  42. 2008.

    The correct answer is 2008. According to the chart, this category shows the largest usage of nuclear energy.

    There is no nuclear energy usage shown before 1980, and all other years indicate less usage than the year 2008.

  43. B. Passage 2 provides a counterargument to the claims made in Passage 1.

    Passage 1 suggests that women from important families and wealthy positions were powerful, while Passage 2 counters that claim by providing examples of wealthy and important women who were swept aside by their respective husbands.

    The correct answer is not Choice (A). The examples given are about two different claims.

    The correct answer is not Choice (C). Passage 2 doesn’t support the theory from Passage 1.

    The correct answer is not Choice (D). There is no problem or solution in this text.

    The correct answer is not Choice (E) because the examples are for two different suppositions.

  44. E. the role of wealth in determining women’s social status for medieval European women.

    Both passages discuss the role that wealth and power played when determining the social status of European women during medieval times.

    The correct answer is not Choice (A). Only Passage 1 alluded to how some women attainted wealth and power — through widowhood.

    The correct answer is not Choice (B). Neither passage suggests that the objective of women during this time period was to acquire wealth and power.

    The correct answer is not Choice (C) because the selection only refers to war as a way to explain how war caused many women to become widows.

    The correct answer is not Choice (D). Neither passage discusses how effective wealth and power were, so much as it discusses the causes and then subsequent effects of it.

  45. C. women’s life situations were a consequence of male attitudes.

    Both passages give examples of how male attitudes about wealth and power directly influenced the lives of individual women, both positively and negatively.

    The correct answer is not Choice (A). Neither passage suggests that women had any influence on men during this time period, other than as an object of wealth and power.

    The correct answer is not Choice (B). The feelings of females were not considered important in the discussion in either passage.

    The correct answer is not Choice (D). Men were not held to a consequence in either passage. They maintained control in both examples.

    The correct answer is not Choice (E). While this would apply to men, the same could not be said of women. So, this statement isn’t the best answer.

  46. E. women had little to no say about their future courses, in regard to marriage.

    The passage says that, while many women might have been left wealthy widows, they were often remarried without their own consent. So, they had little to no say about their future circumstances.

    The correct answer is not Choice (A) because the author states that women were often remarried without their own consent.

    The correct answer is not Choice (B) because the author gives no suggestion that men only married for wealth and power.

    The correct answer is not Choice (C). Passage 1 does not give this example.

    The correct answer is not Choice (D) because this is the premise of Passage 2.

  47. A. introduces a new concept

    In Passage 2, a new concept is introduced, the concept of a potentially powerful woman. Although the treatment given her is the same, this idea is different than in Passage 1 where women were forced to rely on men for attainment of wealth. Here, the queen could have her own power and wealth independent of a husband, but often still suffered the same consequence.

    The correct answer is not Choice (B). No concessions are made that showed the original theory didn’t hold up.

    The correct answer is not Choice (C). There are no digressions from topic.

    The correct answer is not Choice (D). The highlighted example doesn’t anticipate an objection. It introduces, rather, a differing viewpoint on the same topic.

    The correct answer is not Choice (E). There is no proposal of a new social hierarchy, for example.

  48. C. rescinded

    “Rescind” means to agree to something, then to take back, or break the agreement. Here, Charles VIII had agreed to marry Margaret of Austria, but then changed his mind and broke the agreement.

    The correct answer is not Choice (A). This is nearly an antonym of the word “repudiated.”

    The correct answer is not Choice (B). “Reprove” means to reprimand or rebuke someone. It is not the best choice here.

    The correct answer is not Choice (D). By breaking the betrothal agreement, Charles VIII didn’t actually marry Margaret of Austria and didn’t then divorce her.

    The correct answer is not Choice (E). “Acknowledged” is not a synonym or antonym of the word “repudiated.” This word choice just doesn’t make sense in the sentence.

  49. D. propose a creative approach for teaching history.

    This passage attempts to challenge teachers to find something that will appeal to a modern student audience, such as the TV show or book series Game of Thrones.

    The correct answer is not Choice (A). The purpose of the passage is a suggestion that history is better learned via new methods, not to underscore the subject completely.

    The correct answer is not Choice (B). No differences are given. The purpose of the passage is simply to suggest a new idea.

    The correct answer is not Choice (C). The passage is suggesting something not necessarily traditional, so it isn’t a defense of traditional teaching methods.

    The correct answer is not Choice (E). No admonishments are given nor is there any indication that anyone would be opposed to such an idea.

  50. D. connections between false facts and teaching opportunities.

    While Game of Thrones might not stick to a strictly historically accurate script, rethinking false facts can actually provide opportunities for more knowledge. In effect, teaching how something isn’t true will also result in positive gains in learning the subject.

    The correct answer is not Choice (A). The example given is one that is shown as important to understand from a pedagogical perspective and is not being attributed to a rookie teaching mistake.

    The correct answer is not Choice (B). This is not a modern cultural truism. This is an ancient cultural truism.

    The correct answer is not Choice (C). The example is still within the discipline of history.

    The correct answer is not Choice (E) because no modern beginning teacher would teach a “flat Earth” theory.

  51. C. parodying a famous scientific event in a student-produced drama

    One point of the passage is that history can be learned from the perspective of differentiating between truth and untruths. Being able to parody a famous scientific event means a student would have the critical thinking skills necessary to distinguish between truth and parody.

    The correct answer is not Choice (A), Choice (B), or Choice (E). This is simply using some aspect of pop culture to learn more about a topic. It doesn’t require a student to correct any misunderstandings about a subject.

    The correct answer is not Choice (D). Assuming a point of view would give a different perspective of a subject, but would not require the higher-order thinking skills referenced in the passage.

  52. A. how women have been subjugated because of patriarchal oppression

    Both paragraphs explain patriarchal oppression in terms of thinking in past times.

    The correct answer is not Choice (B). No explanation is given as to why the oppression has been tolerated.

    The correct answer is not Choice (C). No part of the passage exemplifies women who achieved glory and honor by rising above patriarchal oppression.

    The correct answer is not Choice (D). An explanation is given of conventional thinking, but no definition.

    The correct answer is not Choice (E). While ideas from the past on femininity are discussed, no explanation is given as to how this idea has changed over time. The reader has to rely on his or her own understanding of the subject in order to do this.

  53. A. powerful women who still could not escape patriarchal ideas of what the ideal female should be.

    All three women are mentioned to show that women were expected to conform to ideas regarding behavior and thought. Politics was outside the range of acceptable thinking for women of the time, even the queens and powerful women of the day.

    The correct answer is not Choice (B) because the women are not mentioned as having set new trends in ways of thinking.

    The correct answer is not Choice (C) because it doesn’t exactly make sense. A historical standard of women? History doesn’t have a standard for how people should behave.

    The correct answer is not Choice (D) because none of these women were models of these qualities. In fact, most broke the mold for being outspoken and forward-thinking.

    The correct answer is not Choice (E) because no examples are given of how women subjugated others, only how women were oppressed by a male-controlled society.

  54. C. a narrow focus on political matters to only include acceptable topics as deemed suitable

    The author states that all matters of politics were seen as distasteful and better left to men.

    The correct answer is not Choice (A). Women’s prime purpose was considered to be a concern with only domestic business and is clearly mentioned.

    The correct answer is not Choice (B). The entire passage suggests all ideas of femininity had to be approved of by the masculine ideas of the time.

    The correct answer is not Choice (D). The passage explains that any obvious sign of womanly intelligence could be evidenced “indoors,” but shouldn’t be seen by the larger, general public.

    The correct answer is not Choice (E). The passage explains that the clearly defined role of femininity was mostly negative in tone and effect.

  55. A. Upper-class women were likely very highly educated and would learn to read and write, as well as speak multiple languages.

    This true fact would not be used to show how women were kept under the thumbs of male oppression as this would show that women were capable of intelligence and independent thought not centered around running a household.

    The correct answer is not Choice (B) because all these examples are centered around domestic business.

    The correct answer is not Choice (C) as this certainly shows a purpose of domestic business.

    The correct answer is not Choice (D) because this example also shows a woman’s circumstances relying on male decision-making.

    The correct answer is not Choice (E). Taking care of others still fits in the traditional and acceptable “female” pursuits of the time.

  56. D. Such oppression permeated the history and culture of the time period.

    The author notes that the thinking of the time influenced politics, as well as books written during and even after the time period.

    The correct answer is not Choice (A) or Choice (B). No comparison is made of femininity then and now.

    The correct answer is not Choice (C). The opposite is true. Matters of politics were certainly influenced as noted by the examples and how they were viewed with distaste.

    The correct answer is not Choice (E). The author doesn’t make or suggest such a claim.

Part 2: Writing

  1. C. behaved cowardly.

    Although “cowardly” ends in “-ly,” it isn’t an adverb. Rather, it’s an adjective. To modify the verb “behave,” you have to use an adverb and not an adjective. To be correct, the sentence might have read, “… felt they behaved in a cowardly way …”

    The right answer is not Choice (A). This is the correct way to show possession of the citizens who belonged to the town.

    The right answer is not Choice (B). “Mayor” doesn’t need to be capitalized. Although this is a title, it is being used more generally here, such as with the word “the” in front of it. Had there been a name after “mayor,” it would have been capitalized.

    The right answer is not Choice (D). Here, the hyphen joins two words that serve as a single adjective before the noun “transportation.”

    The right answer is not Choice (E) because there is, in fact, an error in the sentence.

  2. B. legislation,

    Since two independent clauses are given, a semicolon is needed after “legislation” instead of a comma. The sentence should read, “However liberally minded Senator Scott was, she couldn’t agree with the proposed legislation; rather, the bill’s premise was sure to cause great furor that couldn’t be supported by either side unbiasedly.”

    The right answer is not Choice (A). Here the hyphen joins two words that serve as a single adjective before the noun “Senator Scott.”

    The right answer is not Choice (C). This would be the correct way to show the superlative of “furor.”

    The right answer is not Choice (D). This is an adverb that modifies “supported.”

    The right answer is not Choice (E) because there is, in fact, an error in the sentence.

  3. A. they would receive

    Since the subject is single (“Neither”), the pronoun should also be singular. “They” is a plural pronoun and is not correct.

    The right answer is not Choice (B). “Even though” is a conjunction and is used correctly here.

    The right answer is not Choice (C). This is a correct subject-verb agreement.

    The right answer is not Choice (D). The word “plethora” should be used with “of” to indicate what there is too much, or an abundance, of.

    The right answer is not Choice (E) because there is, in fact, an error in the sentence.

  4. A. a member

    Since the description applies to multiple students (“all of the students”), there should be multiple members who wanted to join the ROTC. However, “a member” indicates only one member, which is incorrect.

    The right answer is not Choice (B). The assembly could have emphasized patriotic duty, and this is represented in a correct way, grammatically speaking.

    The right answer is not Choice (C). The preposition “except for” is used correctly here, not including the drama club members.

    The right answer is not Choice (D). The subjective pronoun “who” is used correctly as the subject of the verb “were.”

    The right answer is not Choice (E) because there is, in fact, an error in the sentence.

  5. B. Georgia

    “Georgia” should have a comma after it because a comma is required after the state name. So that part of the sentence should read this way, “… Savannah and Atlanta, Georgia, …”

    The correct answer is not Choice (A). An acronym can be placed in parentheses after the word or phrase it represents.

    The correct answer is not Choice (C). Commas should be placed after cities, states, and counties in sentences.

    The correct answer is not Choice (D). You only need to capitalize “bachelor” and “master” when giving an official name of a degree, not the general degree.

    The right answer is not Choice (E) because there is, in fact, an error in the sentence.

  6. D. fixing it.

    This is a vague pronoun reference. What, exactly, did Maude fix—the wiring or the lamp? The pronoun doesn’t have a clear antecedent and is considered faulty.

    The correct answer is not Choice (A). This is the beginning of an introductory clause and is correct.

    The correct answer is not Choice (B). While the wiring might be defective, this part of the sentence is correct and works.

    The correct answer is not Choice (C). “Vintage” can correctly modify the “lamp.”

    The right answer is not Choice (E) because there is, in fact, an error in the sentence.

  7. B. no increase

    This sentence falls into the double negative category. When any sentence contains more than one negative, the meaning is unclear to the reader. Because this sentence has both “hardly” and “no,” one of them must be eliminated for clarity.

    The right answer is not Choice (A). Because “Philippines” ends with an “s” and is a proper noun, possession can be shown with just an apostrophe at the end.

    The right answer is not Choice (C). There is no error as the verb “does” agrees with the subject.

    The right answer is not Choice (D). This is a correct way to show possession of “world.”

    The right answer is not Choice (E) because there is, in fact, an error in the sentence.

  8. C. has been known

    Here, the tense doesn’t agree. During his time, Alexander had been known as a radical and rebellious leader.

    The right answer is not Choice (A) or Choice (B). Both tenses agree with their subject.

    The right answer is not Choice (D). Again, the wording here agrees in number and tense.

    The right answer is not Choice (E) because there is, in fact, an error in the sentence.

  9. A. tremendously

    This is a misuse of an adjective. The word “tremendous” is being used here as an adverb, but the adjective form is needed to modify “upheavals.”

    The right answer is not Choice (B). This is the correct word usage.

    The right answer is not Choice (C). The word is a compound noun that is written as a single word without a hyphen.

    The right answer is not Choice (D). Western Africa is a subregion of Africa with defined inclusion of certain countries and is a proper noun.

    The right answer is not Choice (E) because there is, in fact, an error in the sentence.

  10. C. her

    This sentence has a pronoun case agreement problem. Here, the pronoun should be the subjective form for Mary, not the objective form. So, this sentence part should read, “… neither Juan nor she… .”

    The right answer is not Choice (A), Choice (B), or Choice (D) because all are correct verb tenses.

    The right answer is not Choice (E) because there is, in fact, an error in the sentence.

  11. D. she wanted

    The sentence is incorrect because it does not have parallel structure. Since the other forms include “-ing,” this verb should as well. So, this part of the sentence should read, “… and wanting to become… .”

    The right answers are not Choice (A), Choice (B), or Choice (C). All these forms are parallel and are error free.

    The right answer is not Choice (E) because there is, in fact, an error in the sentence.

  12. E. No error

    There are no errors in the sentence in terms of either word choice or punctuation.

    The right answer is not Choice (A). Since the sentence begins with an introductory clause, a comma is appropriate here.

    The right answer is not Choice (B). The infinitive verb is used appropriately here.

    The right answer is not Choice (C). Since the word “North” is capitalized, a reader should understand that this likely refers to a specific part of Boston. So, both parts would be proper nouns and should be capitalized.

    The right answer is not Choice (D) because this is the correct past tense of the verb “burst.”

  13. C. dissimilarity between

    The word “dissimilarity,” because it shows a comparison, should either have a “to” after it or should have another reference in the sentence, such as “… the dissimilarity between her twin sister and herself was unnerving.”

    The right answer is not Choice (A). This is the correct tense for the verb form “to bear.”

    The right answer is not Choice (B). The word “resemblance,” like the word “dissimilarity,” needs a “to” here.

    The right answer is not Choice (D). This is an appropriate word choice and usage.

    The right answer is not Choice (E) because there is, in fact, an error in the sentence.

  14. A. Mowing

    This is an example of a dangling modifier. It’s unclear who was mowing the front lawn. However, the way the sentence is written, it appears as if the snake were mowing the lawn and sunning itself. Clearly, this is not correct.

    The right answer is not Choice (B). This modifier identifies which snake became alarmed — the one sunning itself.

    The right answer is not Choice (C). This reflexive pronoun is referring back to the snake and is appropriate.

    The right answer is not Choice (D). This past tense form of the verb “to slink” is correct usage.

    The right answer is not Choice (E) because there is, in fact, an error in the sentence.

  15. B. adverse

    This is an incorrect use of the word “adverse” which means “unfavorable.” Instead, the word “averse” is needed here, which means “opposed to.” The two words are commonly confused with one another.

    The right answer is not Choice (A). This word doesn’t need to be capitalized because it is a general use of the term and not used as a proper noun with the names of the senators.

    The right answer is not Choice (C). The idiom “put forth” is used correctly here.

    The right answer is not Choice (D). The preposition “of” goes with the word choice “possibility.”

    The right answer is not Choice (E) because there is, in fact, an error in the sentence.

  16. D. among.

    The word “between” should be used instead. When referring to distinct and individual items, “between” is used; “among” is used to refer to general things. Because specific colleges are named, this is a mistake.

    The right answer is not Choice (A). This sentential adverb is used correctly.

    The right answer is not Choice (B). The number refers to both the ACT and SAT verbal reasoning portions.

    The right answer is not Choice (C). This is a correct use of the adjective “fortunate.”

    The right answer is not Choice (E) because there is, in fact, an error in the sentence.

  17. B. is known

    This part of the sentence has an inappropriate verb tense shift. Because the statement uses other verbs and situations in the past, this part of the sentence cannot be in the present.

    The right answer is not Choice (A). This is a correct use of the subjective pronoun “who.”

    The right answer is not Choice (C). Again, all tenses should be in the past.

    The right answer is not Choice (D). Here, the word “traveling” is an adjective and modifies the word “hypnotist.”

    The right answer is not Choice (E) because there is, in fact, an error in the sentence.

  18. C. home,

    This is a run-on sentence. The two sentences should be joined here either by a semicolon or by a period and new sentence.

    The right answer is not Choice (A). While the two names are used interchangeably, putting the Latin form in italics is not incorrect.

    The right answer is not Choice (B). “War” does need to be capitalized because it is referring to a specific war and is a proper noun.

    The right answer is not Choice (D). “Hence” typically begins a sentence. It is an appropriate usage here and is often followed by a comma.

    The right answer is not Choice (E) because there is, in fact, an error in the sentence.

  19. C. are

    This is a subject-verb agreement problem. The subject (“state”) is singular and should have a singular verb (“is”).

    The right answer is not Choice (A). The word “state” is used here as a general term, not as a proper noun.

    The right answer is not Choice (B). Any whole number from twenty-one through ninety-nine is hyphenated when written out.

    The right answer is not Choice (D). This part of the sentence contains no usage error.

    The right answer is not Choice (E) because there is, in fact, an error in the sentence.

  20. A. than the symbol of the clown itself

    Here, “itself” is correctly used as an intensive reflexive pronoun that serves to emphasize its antecedent, clown.

    The right answer is not Choice (B). An intensive pronoun appears after the noun it is emphasizing, which is “clown” in this example.

    The right answer is not Choice (C). “Symbol” is not the word that “itself” is referring to.

    The right answer is not Choice (D). Intensive reflexive pronouns are used to draw attention to a word or phrase. So, while intensive pronouns are not essential to the basic meaning of a sentence, leaving it out here changes the tone of the statement or the author’s purpose, which was to draw attention to the relationship between the two words.

    The right answer is not Choice (E) because this doesn’t sound right. The pronoun occurs after the noun or pronoun it refers to, not before.

  21. A. who is considered the father of architecture, ichnology, and paleontology, likely also invented

    This clause must be separated by commas. The sentence itself is correct for subject-verb agreement in terms of tense.

    The right answer is not Choice (B). “Leonardo Da Vinci” is the subject. Adding an additional pronoun is incorrect.

    The right answer is not Choice (C). This choice is incorrect subject-verb agreement.

    The right answer is not Choice (D). Again, this would be incorrect subject-verb agreement.

    The right answer is not Choice (E). Removing the “who” from the phrase turns this into a verb phrase. A verb would not be separated from the subject by a comma.

  22. D. and the ability to secure and manage energy and electricity while away from home, all the while saving

    The clause at the end of the sentence is correct, but should be separated by a comma instead of a semicolon.

    The right answer is not Choice (A). A semicolon should only be used to separate independent clauses. The clause at the end of this sentence is dependent.

    The right answer is not Choice (B). To avoid confusion, a comma is used to set off clauses and phrases.

    The right answer is not Choice (C). Adding an “s” to the end of “saving” changes it from a verb to a noun and doesn’t make any sense.

    The right answer is not Choice (E). Changing the verb tense doesn’t correct the punctuation problem.

  23. E. HBO’s Game of Thrones has reached an international fan base and has even been acclaimed

    The sentence is simply missing the present perfect “has.”

    The right answer is not Choice (A) because of the missing word “has.” Leaving it out creates an unbalanced and incorrect sentence.

    The right answer is not Choice (B). This change creates a meaning change. There is no indication that an international fan base is because of the acclaim by critics.

    The right answer is not Choice (C). This change causes an incorrect shift from active voice to passive voice, changing the recipient of the action.

    The right answer is not Choice (D). This change also causes a change in meaning and suggests that the critics’ acclaim is due to the international fan base.

  24. A. The International Ice Patrol tracks icebergs globally, a practice that allows the monitoring of their origins and other ocean processes.

    The sentence is correct. The focus is on the pronoun “their” and what it refers to. This choice makes it clear that the icebergs are being monitored.

    The right answer is not Choice (B). Here, it seems as if the International Ice Patrol is being monitored because “their” is referring to that organization instead of the icebergs.

    The right answer is not Choice (C). Clearly a pronoun is needed to avoid word repetition.

    The right answer is not Choice (D) or Choice (E). Removing the pronoun and rearranging the sentence changes the voice and makes it seem as if the International Ice Patrol has somehow gotten permission to do this task.

  25. C. to protect against danger for those undertaking

    This sentence suffers from an error in parallel structure. Because “to safeguard” was used, the same structure needs to be after the conjunction. So, it would be “to protect.”

    The right answer is not Choice (A) or Choice (B). Neither choice corrects the incorrect parallel structure problem.

    The right answer is not Choice (D) or Choice (E). While the parallel structure has been corrected here, there is no reference for the pronoun “they.”

  26. B. If visitors were to travel to the most eastern part of the western world, they would find themselves at Ilomantsi, Finland, a location sharing a 60-mile border with the Russian Republic of Karelia.

    This is the clearest, most correct, and most succinct of the many choices.

    The right answer is not Choice (A). This answer incorrectly uses a colon to separate the last phrase from the rest of the sentence.

    The right answer is not Choice (C). While most of the information is correct, the exact location isn’t the border. This section puts most of the emphasis on the border instead of Ilomantsi.

    The right answer is not Choice (D). Again, the emphasis is on the border, not the exact location. This nuance affects the meaning.

    The right answer is not Choice (E) because this is not correct. Again, the location for the easternmost part of the western world isn’t the border, but Ilomantsi, Finland.

  27. D. some recounting

    Of all the choices, this one best corrects the structure of the sentence so that the conjunction joins parallel structures.

    The right answer is not Choice (A). The conjunction “and” doesn’t join parallel structures.

    The right answer is not Choice (B). A wooden pole doesn’t have the ability to recount legends. This is just a poor word choice.

    The right answer is not Choice (C). The word “and” isn’t needed and doesn’t join two events or things.

    The right answer is not Choice (E) because, again, the conjunction doesn’t join parallel structures.

  28. B. as well as

    “As well as” connects the two unique physical features of the fish — the growth that hangs from its head and the lantern located on its fin.

    The right answer is not Choice (A) or Choice (C). The way they are written makes it seem as if the growth includes the lantern, which isn’t correct.

    The right answer is not Choice (D). Since “and” is already being used to indicate two features of the growth (location and purpose), adding another here would be confusing to the reader.

    The right answer is not Choice (E). This isn’t the best joining word for these two features.

  29. C. Amy filled her mind with thoughts of a long and pleasurable night of relaxation in a tub with a good book.

    Since the pronoun “her” has been used, it must refer to Amy, not Amy’s mind. So, this is the best choice.

    The right answer is not Choice (A), Choice (B), or Choice (D). Who is finishing her calculus? Amy’s mind? The pronoun references are unclear with these choices.

    The right answer is not Choice (E). While grammatically correct, it’s long and awkward.

  30. D. had trained for months for the physical requirements

    She was training for the physical requirements. And, the tense needs to be past since the event already occurred.

    The right answer is not Choice (A). The tense is wrong in this choice, and she wasn’t trained in the requirements. This is just an odd word choice.

    The right answer is not Choice (B). Again, the tense is wrong.

    The right answer is not Choice (C). This suggests the training reflects a state of being rather than an exclusive and timed event.

    The right answer is not Choice (E) because, again, the tense is wrong.

  31. A. No change.

    The word “while” is an appropriate choice to show the transition of such changes over time.

    The right answer is not Choice (B). This word choice means “but” and doesn’t make the most sense here. Also, this word generally separates independent clauses, as with a semicolon, or introduces a new thought in a separate sentence.

    The right answer is not Choice (C). The information about today’s society is not an example from fifty years ago. These are two different things.

    The right answer is not Choice (D). This word choice means “so” or “therefore.” The relationship between these two ideas is not one of cause and effect, but change over time.

    The right answer is not Choice (E). This idiom shows two opposite ways of thinking about something. The ideas here are not opposite ways of thinking, but changes over time.

  32. A. No change.

    Although the ideas are conjoined, these two can exist as separate and independent sentences. Of all the answer choices, this one is the best and most correct.

    The right answer is not Choice (B). It’s unclear who “they” are.

    The right answer is not Choice (C). In this example, it seems as if the product is making the goods.

    The right answer is not Choice (D). Again, “they” has no reference and the tense is wrong.

    The right answer is not Choice (E) because there is a difference between goods and the goods. In this example, no specific goods are mentioned, so the article isn’t needed.

  33. C. immediately after Sentence 6

    This placement makes the most sense. The previous sentence refers to goods, ideas, and information, and this sentence gives examples of both goods (food) and ideas and information (patterns of daily communication).

    The right answer is not Choice (A). Sentences 3 and 4 are connected by topic, so this choice wouldn’t make sense.

    The right answer is not Choice (B). Sentence 4 refers to goods, and this sentence is about a different topic.

    The right answer is not Choice (D). Sentence 7 talks about the price of such technology, and this sentence would not elaborate on that idea.

    The right answer is not Choice (E). Sentence 9 is a completely new topic, and this sentence would not further that thought in any way.

  34. D. Replace “for all practical purposes” with “in effect.”

    The words “in effect” means this is one resulting effect of such actions.

    The right answer is not Choice (A). It isn’t actually practical for the government to spy on its citizens, so Choice (D) makes the most sense.

    The right answer is not Choice (B). The sentence is connected to the one before it with a similar idea.

    The right answer is not Choice (C). Here, the government is seen as the entity, not necessarily the people who make up the government. For example, lawmakers are not responsible for this effect of technology.

    The right answer is not Choice (E). “Snoop” is a gentler version of the same idea, but the stronger “spy” is called for here.

  35. C. Replace “prompt” with “instantaneous.”

    Something can’t be almost prompt. It can, however, be almost instantaneous. This change makes the most sense with the topic.

    The right answer is not Choice (A). The intent is that the information is being actually sold, not simply offered as a service.

    The right answer is not Choice (B). The data includes more than just statistics.

    The right answer is not Choice (D). The pronoun “it” directly refers to the idea presented in the previous sentence.

    The right answer is not Choice (E). This sentence is needed because it further elaborates a point made.

  36. E. Replace “provided” with “so long as”.

    Although the two can be used interchangeably, “provided” doesn’t make as much sense as “so long as.” This would be the best change to make.

    The right answer is not Choice (A). The change would add an additional subject and make the sentence confusing and incorrect.

    The right answer is not Choice (B). This revision would change the nuance of the sentence, from something that is simply advantageous to something that has a monetary value.

    The right answer is not Choice (C). “Everyone” doesn’t just refer to the major shareholders; it would literally benefit every person involved. Besides, who would the major shareholders be?

    The right answer is not Choice (D). This isn’t a needed change and wouldn’t drastically improve the sentence.

  37. E. location of the publisher.

    The publisher is located in Lexington, Kentucky.

    The right answer is not Choice (A). The business address of the author is not given in a citation.

    The right answer is not Choice (B). The scholarly affiliation, or name of the publisher, is the second reference of Kentucky, the University Press of Kentucky.

    The right answer is not Choice (C). There is no indication that specific games are being discussed as being more philosophical than others.

    The right answer is not Choice (D). The book itself is the collected philosophies of this topic.

  38. D. A script of a speech that she wrote and delivered to the UN while running for president.

    A primary source is a first-person account written by the person who participated in and witnessed events. A text written by the person about his or her experiences is the only example that is correct here.

    The right answer is not Choice (A) or Choice (B). Both choices are researched and referenced, meaning the information comes from another source.

    The right answer is not Choice (C) or Choice (E). These two choices may give information that is more original, but they are still considered secondhand sources. In this example, only the words of Hillary Rodham Clinton are considered firsthand.

  39. A. models showcasing the fashion on runways

    This topic is somewhat related in that both are about fashion. However, this topic doesn’t narrow the article focus.

    The right answer is not Choice (B), Choice (C), Choice (D), or Choice (E). All of these are on topic and would narrow the focus of the article.

  40. B. Abstract

    Abstracts appear at the beginning of a text and are used to help the reader quickly see the main points and purposes of the text.

    The right answer is not Choice (A). A bibliography is a list of works referred to in a scholarly work. Bibliographies are usually printed as an appendix, at the end.

    The right answer is not Choice (C). An outline is a general description of the plan of a work. It is usually not seen or printed with a text, but is used by a writer when organizing and completing the text.

    The right answer is not Choice (D). A preface is found in a book, not a research paper. However, it does appear at the beginning and often gives the subject, scope, or aim of the work.

    The right answer is not Choice (E). While appearing at the beginning of a work, the table of contents does not provide a summary. It simply shows the organization of the work.

Argumentative Essay

Take a look at the following essay written in response to Question 41 in Chapter 14. To score your own essay, flip back to Chapter 11, where you can find a checklist to help you evaluate your own writing.

  • It is hard to imagine a day without technology to accomplish even the simplest of things. An evening without power after a storm is the closest most of us get and those long hours with no quick access to electricity make the Little House on the Prairie book series seem more like a work of horror than historical fiction. Asking a young person to imagine a day without their cell phone would probably reveal it would be impossible to go an hour without those important connections to a cyber world. Have we, though, as a technologically-savvy generation become too dependent on technology? Yes, our generation is too dependent on technology and the negative aspects of the isolation caused by technology should be a cause for serious alarm.
  • Many would say that the word “dependence” is not nearly critical enough, that this generation has passed dependency and has, instead, become addicted to technology. Much research has been done to show that overuse of the Internet and overuse of technology leads to addiction. Those who overuse the Internet have the classic symptoms of compulsive behavior that is similar to gambling addiction and compulsive shopping. People who become preoccupied with the Internet and are unable to control their usage should be labeled “addicted”, in the same way that people who are preoccupied with alcohol or drugs are labeled as addicts. Such addiction ruins lives and marriages, from the grandmother who spends her retirement savings by shopping online to the lonely teenager who visits online porn sites and resists interacting with the “real”, outside world. This isolationism is translating into youth who never go on a date and only communicate with virtual mates online. MTV’s Catfish, a reality show that brings together couples who have interacted, often disastrously, solely through technology, is this generation’s Dating Game.
  • The rise of such “real-time” technologies, like Internet relay chat, live chat, and multi-user domains in virtual worlds are creating a generation of people who struggle to interact in social situation when confronted with a live person. This new technology will likely cause rising incidents of a new population, youth, who would regularly choose to use this technology and completely avoid interaction making interviewing for jobs, establishing stable relationships, and even simple friendships next to impossible.
  • Many would argue that technology is necessary for business and this argument is overgeneralizes American technological success. While technology can certainly improve work results, an overreliance on that technology can also result in technology addiction. The overreliance of something as simple as mobile email use can cause feelings of perceived work overload and increased examples of work-family conflict. The reliance of having email always handily available does not actually increase work performance; rather, it can decrease performance because of feelings of inadequacy. Checking emails and social media constantly throughout the day to see what “hit” can leave users feeling a disconnection from the world while amid a family or social gathering. Clearly, technology has skewed our vision of what the term “communication” means, a back and forth dialogue between people. Instead, technology has created a generation of introverts who only seek ideas and information and never hone the practice of imparting or conveying messages.
  • Today, we are dependent on technology for every aspect of our lives, from the way we get our news to way we pay our bills. The first thing most people reach for in the morning is the phone, either to silence an alarm or to fast-check what event might have happened during the hours allotted to sleeping. Breakfast is cooked with a microwave, the laptop is packed for a busy workday, and cars or buses with computerized chips drive us to a location where we once again engage in a virtual world. This discussion might be better argued with, “Can we even live without technology?” What part of you day is isolated from any type of technology? Realistically, perhaps only sleeping on a mattress in a darkened room is the only thing that remains.
  • While computers and technology are staples of the modern world, a balance must be reached between what is needed to enhance life and what becomes life-draining as far as time as resources go. The distinction between use and addiction is one that can be debated, but there is no doubt technology has taken over our lives.

Using the checklist in Chapter 11, here’s why this essay would receive a score of 6.

Introduction: The introduction captures the reader’s attention in two ways. First, it forms a connection with the reader. By asking us, the reader, to imagine a day without technology, a connection is made between the reader and the writer. For most of us, going without technology for even one day is impossible to think about … and yet, we also know this isn’t necessarily a good thing. This connection with the reader is persuasive because the writer has already capitalized on common ground. Another attention getter is the reference to Little House on the Prairie. Since it was a popular show and book, the humorous point about it being a “work of horror” because of the lack of technology is appealing and starts the argument out in a way that doesn’t alienate the reader.

The middle paragraphs: The writer keeps this moment flowing by continuing to appeal to a modern reader with references to reality shows and other uses of technology. The points made are valid and presented in such a way that even the most die-hard technophile would agree. The author also gives counterarguments, which further appeals to the reader because they are reality-based in the daily struggle with technology.

Conclusion: The conclusion sums up the argument, restates the thesis, and brings closure.

There are grammatical and syntactical errors. So while the essay isn’t perfect, it doesn’t have to be. The checklist doesn’t require perfection, only a high degree of competence. Here, the writer’s position is clear and well organized. There are strong supporting details and excellent use of grammar with a particular purpose in mind — appealing to an audience that relies on technology.

Source-Based Essay

Take a look at the following essay written in response to Question 42 in Chapter 14. To score your own essay, flip back to Chapter 11 and use the checklist to help you evaluate your own writing.

  • Both passages address the topic of animal rights with a consideration of what it means to have a “right.” By human standards, most Americans probably would first call to mind constitutional rights — the right to life, liberty, and the pursuit of happiness given by our forefathers in the Declaration of Independence. These “rights,” though, were lawfully and rationally designed for the purposes of government. Certainly the U.S. is not alone as all countries have laws that propose much the same thing. These rights were granted to humans by other humans and the government has the duty to protect those rights. So, then, a right is anything that is morally good and justified as being worthy. A look at any major religion or culture across the globe from the beginnings of time will show that such rights are not new concepts, but are considered basics. The protection and reverence for life is a basic rule that humans, and animals, hold sacred.
  • “Animals hold life sacred?”, you may be skeptically asking. The well-known phrase “protecting young like a mother bear” settles that question, but there are other instances of this sanctity of life in the animal kingdom. Mothers who wail and yearn for missing or dead young, colonies who spend lifetimes protecting one another, paired mates raising a family, and chimpanzees and elephants who bury their dead — all these are instances of the value of life placed on different species and are not just human concepts. A counterargument might point out the obvious kill-or-be killed disposition found among species, both in the wild with lions and bears and in every backyard with mice and cats. However, the same could be said of man with his countless wars. Protecting the valued lives of ones held closest or for survival is not casting off the value of life, but another example of the extreme reverence placed upon it.
  • Singer insists that accepting this viewpoint of animals, only regarding the relationship they have with humans, is flawed thinking and he likens it to the rationalization a racist brings to those acts and thoughts — the idea that some other thing can only be valued in comparison to me (Singer 2014). If we only look at an animal regarding how it will enhance or improve our own quality of life, we diminish that which we all hold sacred — the value of life itself. For at what point does any one human life weigh more than another? The judgment should not be on which life is more valuable. All lives, according to Singer, should be afforded that honor.
  • Regan also points out that the way we view animals, as an entity completely separate from ourselves is inherently wrong. The assumption that animals are here for our own pleasure and enjoyment, as “resources,” is a fundamental flaw in our reasoning (Regan 2014). Humans misunderstand the purpose for animals as a core concept because we can only understand the value in other things in relation to the value it brings to us. Once we start to view animals as something separate from our own “self aware[ness],” then we can better understand that an animal has its own separate value and a right to life (Regan 2014). Animals exist not to bring us pleasure, but for their own sake.
  • Regan uses the term “inherent” to qualify the value of life — both human and animal. This term insists that the value of life, then, is something that is essential to life itself (Regan 2014). This “inherent” quality emphasizes that the understanding of life value is one that is inborn in all life forms; it is an essential and natural instinct and one that has not been learned by humans over time through discourse and discussion. The conclusion, then, is that humans do not decide the value of life and therefore, should not be taking life from animals.
  • Of course, the larger concept is what to do about such gleanings? Should man no longer have pets? Should we all become a vegetarian or fruitarian, eating only what nature can produce? Would the next discussion be directed to the nuances of a vegan or vegetarian lifestyle? Neither author proposes what to do with after accepting the proposed viewpoint and perhaps that acceptance is enough. To understand that animals should not be cruelly abused or used for sport might just be enough of a starting point for animal activism. Both passages promote the basic premise for animal rights — an understanding that all animals, human and other, are entitled to possession of their lives and that these interests should be affording respect and consideration.

Using the checklist in Chapter 11, here’s why this essay would receive a score of 6.

Introduction: The introduction states the thesis right away and finds the common ground between the two readings. Comparing the ideas in the reading is crucial to this type of essay. Logical arguments will always win out over a great opening statement, a catchy title, or engaging lead. You can never go wrong presenting your case up front with a clear thesis. Right away, the reader can anticipate the rest of the essay.

The middle paragraphs: These paragraphs stick to the topic and support the thesis. It’s important to give strong and specific evidence to support your points. Here, the writer gives strong examples from both readings with exact quotes that clearly connect to the thesis. And, they’re correctly cited.

Conclusion: All well written essays should end with some kind of call to action, and this one is no different. While the reader isn’t encouraged to become a vegetarian overnight, the essay has given the reader something to think about.

Again, this essay isn’t perfect. However, a high degree of competence is displayed with evidence of a presentation of a thesis, discussion of the argument, and then conclusion of the main points. Using strong and specific language with sentence variety is always the key to moving your essay up the checklist.

Part 3: Mathematics

  1. A. 43

    Two interior angle measures are given for the top triangle, and that is enough information for finding the third angle measure. The sum of the interior angle measures of every triangle is images. You can add the two given angle measures and subtract their sum from images to get the third angle measure.

    images

    Since that angle measure is images, the upper left angle of the second triangle down is also imagesbecause those are vertical angles, and vertical angles are always congruent. You can find the number of degrees in the third angle by subtracting the sum of 49 and 47 from 180.

    images

    Since that angle is images, the top angle of the bottom triangle is also images since those are vertical angles. The sum of images and images is images You can subtract 137 from 180 to find the number of degrees in the third angle, and that is the value of u.

    images

    The measure of the bottom right angle of the bottom triangle is images, so the value of u is 43.

  2. B. 8.9

    You can use a variable, such as x, to represent the minimum score and write an equation with it. The mean of a set of data is the ratio of the sum of the numbers to the number of numbers. You can write an equation in which that ratio is set equal to the minimum mean score of 9.2, and solve for the variable.

    images

    A score of 8.9 for the final event would cause Joey’s mean score to be 9.2, and anything above 8.9 for the final event would give Joey a mean score that is higher than 9.2. Therefore, 8.9 is the lowest score Joey can receive for the final event to get a mean score of at least 9.2 for the overall competition.

  3. A. 4

    First, multiply the two fractions by putting the product of the numerators over the product of the denominators.

    images

    Then, simplify the result by dividing the numerator and denominator by their greatest common factor.

    images

    The numerator in the simplified form of the product of images and images is 28. To find the factors of 28, write its prime factorization, which is an expression of 28 as the product of all prime numbers.

    images

    The factors of 28 include only those prime numbers and all of the products of combinations of them, and also 1. The only choice that is such a number is Choice (A). It is the product of images.

  4. C. images

    The problem does not give enough information for the numerical value of r to be determined, but you can find the value of r in terms of the other variables by getting r by itself on one side of the equation.

    images

    You can multiply that rational expression, which represents the value of r, by 3 to find the value of 3r.

    images
  5. B. 2014

    The line graph drops from 2013 to 2014, and the drop is approximately 400. The only other decreases are from 2015 to 2016 and from 2016 to 2017, and both drops are clearly well under 400. The changes from 2012 to 2013 and from 2014 to 2015 are increases.

  6. D. 36

    This system has three variables but only two equations. However, the value of z is given, so you can substitute the value of z in for z in both equations and get a system of two equations with two variables. That is enough information for you to solve the system. The cube of 3 is 27, so the value of z is 27.

    images

    Next, subtract 27 from both sides so each equation will have only one constant. That can be done later, but the path to solution might be clearer if you do it at this point.

    images

    Since the coefficients of y are opposites, you can add the two equations to get rid of y. That will result in an equation with one variable. An equation with one variable can be solved, if the equation has a solution.

    images

    Since x has a value of 4, you can substitute 4 in for x in any equation used so far and have an equation with one variable. Then you can solve the equation for y because the equation will have only one variable.

    images

    The value of x is 4, and the value of y is 9. The product of 4 and 9 is 36, so the value of xy is 36.

  7. D and E. The survey method is NOT a proper procedure for determining the most popular political party affiliation at the university because the group surveyed is Libertarian and would work as a sample that only or mostly represents Libertarian Party affiliation. The survey method is NOT a proper procedure for determining the most popular political party affiliation at the university because political parties other than the Libertarian Party would not be likely to be represented by the sample.

    For a sample to represent a larger group properly, it cannot be a biased sample. In other words, it has to have reasonably close to accurate portions of the different categories that are represented in the data that is collected. The members of the Campus Libertarians are all or nearly all members of the Libertarian Party, so the portions of other political party members at the university would not be correctly represented. Therefore, the survey method described in regard to this question is not proper because the sample is not accurately representative. That makes Choices (D) and (E) correct. Those choices make basically the same point from different angles. The reasoning described here also makes Choices (A) and (C) incorrect. Both of those choices say that the procedure is proper, which it is not. By the reasoning explained here, the rest of the statements for Choices (A) and (C) are also false. Choice (B) is false because it says that the Campus Republicans would be representative of the entire campus. That is false. No campus group representing a political party is politically representative of the entire campus.

  8. D. 34

    The first step is to divide 100 by 3 to get an initial picture of how many sets of three balls are needed.

    images

    That is precisely how many sets of 3 balls are in 100 balls. However, an extra issue is involved in this scenario. The situation is not described as having any sets of tennis balls other than those that contain 3 tennis balls. There is no mention of containers with 1 ball each. Because of that, an entire container of three balls is necessary beyond the first 33 containers. That will make a total of 102 balls, but it is the only way to cover 100 balls. That means 34 containers are necessary.

  9. E. 1,000

    The diameter of a sphere is twice its radius. If you know the radius of a sphere, you can double it to get the diameter. The formula for the volume of a sphere is images. You can substitute the known values of variables in for the variables and solve for r, which represents radius.

    images

    The radius of the sphere is 5, so the diameter of the sphere is 5(2), or 10. The cube of 10, or images, is equal to 1,000. Therefore, the cube of the number of yards in the sphere’s diameter is 1,000.

  10. B. –11

    There are various methods for determining answers to algebra questions like this one, and most of them are beyond the level of what is covered by the Praxis Core. What we suggest for multiple-choice questions like this one is that you plug in choices until you find the one that works. The only choice you can put in for j and get –478 for the value of the quadratic expression is Choice (B).

    images
  11. B. images

    First, determine the value of w by getting w by itself on one side of the equation.

    images

    Next, find 34.5 percent of 548. You can put 34.5 percent in decimal form and multiply it by 548. To put 34.5 percent in decimal form, drop the percent symbol (%) in 34.5% and compensate for that by moving the decimal point two places to the left.

    images

    Next, add 189.06 to 548 to find the result of increasing 548 by 34.5 percent.

    images

    That is the number you need to express in scientific notation. To do so, move the decimal point to where only one digit is before it. That is two places to the left. To compensate for moving the decimal point two places to the left and thereby getting a smaller number, multiply the result by 10 with an exponent of 2.

    images
  12. C. 6.5 inches

    First, determine Gracyn’s actual driving distance. She drove 7.8 miles to a store and drove the same distance on the way home. The total distance is therefore images, which is 15.6 miles. Since one inch on the map represents 2.4 real-world miles, you can divide 15.6 by 2.4 to determine the number of inches covered on the map.

    You could also set up a proportion and solve for the unknown. You can use a variable to represent the number of inches of map covered. The proportion should contain two fractions set equal to each other, with each fraction representing inches to miles or miles to inches. Either type of ratio will work, but both ratios must have the same format.

    images

    Gracyn moves her pen 6.5 inches.

  13. D. images

    All of the choices are in terms of b, so solve for a in terms of b so that you can substitute the value in for a in the rational expression in the question.

    images

    Since images, you can put images in for a in images.

    images
  14. B. images

    This question requires number sense. Specifically, it calls for ballpark knowledge of the measurements of common objects. The mean of a set of numbers is the set’s average. The average tail length of house cats is 1 foot. You are not required to know facts like that, but you are required to have enough knowledge of common objects and measurement principles to approximate average measures of common objects and identify what measurements are far from them. The choices other than Choice (B) are nowhere near one foot. They are all outrageous. A foot is images of a yard, so the correct answer is Choice (B). Choice (A) is a quarter of a foot. Choice (C) is equal to 5 feet and 4 inches. Choice (D) is 7 feet.

  15. D. (–2, –3)

    The coordinates of the vertices of the triangle in the diagram are (–3, 1), (3, 7), and (8, –1). You can determine the coordinates of the vertices of the triangle after the rotation by multiplying all the vertex coordinates of the original triangle position by –1. That is the proper method for determining new points after any images rotation about the origin on the coordinate plane, whether the rotation is clockwise or counterclockwise. The resulting position is the same either way. Multiplying a number by –1 is the same as simply changing its sign, so the coordinates of the vertices of the triangle after the rotation are (3, –1), (–3, –7), and (–8, 1). You can draw a rough sketch of a triangle with those vertices on the coordinate plane and see which choice is clearly on the interior of the triangle in its new position and which choices are clearly not on the interior.

    image

    The point (–2, –3) is on the interior of the triangle in its new position. The other choices clearly are not.

  16. C. 8

    The first sentence describes an equation. You need to translate the equation from English to math language. You can use a variable to represent the lower integer, and another algebraic expression involving that variable to represent the higher integer. Because consecutive even integers are always 2 apart, you can call the lower integer x and the higher integer images. After you write the equation, you can solve it.

    images

    Since the value of x is 6, the lower integer is 6. The higher integer is 2 more than that, so it is 8.

  17. 4

    First, determine the least common multiple of 8 and 10 and the greatest common factor of 84 and 72. To find the least common multiple of 8 and 10, you can write the first few or so multiples of 8 and 10 and find the lowest number that is a multiple of both. Once you find the lowest multiple of 10 that is also a multiple of 8, you can stop writing the multiples of 10 because that is the least common multiple.

    • Multiples of 8: 8, 16, 24, 32, 40, 48, 56, 64
    • Multiples of 10: 10, 20, 30, 40

    You can stop at 40 for multiples of 10 since 40 is also a multiple of 8. The least common multiple of 8 and 10 is 40.

    Now, find the greatest common factor of 84 and 72 by listing all the factors of both. You can use the prime factorization of each to find their factors. Every factor of a whole number is the product of a combination of multiplying prime factors, except 1 is also a factor of every whole number.

    • Factors of 84: 1, 2, 3, 4, 6, 7, 12, 14, 21, 28, 42, 84
    • Factors of 72: 1, 2, 3, 4, 6, 8, 9, 12, 18, 24, 36, 72

    The greatest number that is a factor of both 84 and 72 is 12, so the greatest common factor of 84 and 72 is 12.

    Thus, what you are looking for in this situation is the greatest common factor of 40 and 12. You can write the factors of each number and look for the greatest factor they have in common.

    • Factors of 40: 1, 2, 4, 5, 8, 10, 20, 40
    • Factors of 12: 1, 2, 3, 4, 6, 12

    The greatest number that is a factor of 40 and 12 is 4, so 4 is the greatest common factor of 8 and 12.

    Note: There are other methods you can use to find least common multiples and greatest common factors. A quicker method for finding the greatest common factor of two numbers is to write the prime factorization of both and line up all instances of prime factors. You can multiply every instance of a number that is a common prime factor. The product will be the greatest common factor of the two numbers in question.

    images

    You can see that there are two matchings of 2 and one matching of 3. Therefore, the greatest common factor of 84 and 72 is images, or 12.

  18. B. 24

    The probability of an event is the ratio of the number of favorable, or qualifying, outcomes to the number of possible outcomes. The probability that the name of a person from Mississippi will be drawn from the hat is a ratio of 15 to the number of people at the board meeting, the number which is the answer to the question. That ratio is equal to images. You can set the two ratios equal to each other in an equation. Because the number of people at the board meeting is unknown, you can use a variable to represent it. Then you can solve the equation, which is a proportion because it involves a ratio set equal to a ratio. When a proportion is true, the cross products of it are equal.

    images

    The number of people at the board meeting is 24.

  19. C. images

    You can solve for a variable in terms of other variables by following the same rules and procedures you follow to solve for a variable in terms of its numerical value.

    images
  20. A. 3.14

    When the order of a group of numbers is not clear, convert the numbers to the same form, such as fraction or decimal. In this case, decimal is probably the best form because using the necessary common denominator would be extremely tedious. You can convert a fraction to a decimal by dividing the numerator by the denominator. In decimal form, it is best to give the numbers the same number of digits after the decimal so they can be most easily compared. However, you should not round up the last digit. You can put dots after it to indicate that the digits continue for numbers in which they do.

    images

    These are those numbers in decimal form, from greatest to least:

    3.1904…, 3.1428…, 3.1415, 3.1400, 3.0714… .

    Therefore, the correct order of the number forms in the question, from greatest to least, is this:

    images, images, 3.1415, 3.14, images

    The fourth number in that order is 3.14.

  21. C. images

    A square is a type of rectangle, so the area of a square is the product of its length and width. Because all sides of a square are congruent, a square’s length and width are the same. Therefore, the area of a square is the square of the measure of any of its sides (that is the basis of the term “squared”), all four of which are the same. Because of that, the square root of the area of a square is the measure of each of its sides.

    images

    Since the square root of 100 is 10, each side of the square is 10 units. The diameter of the circle is the same as the measure of each side of the square, so the diameter of the circle is also 10 units. The area of a circle is equal to images, for which r is the radius of the circle. The radius of a circle is half the circle’s diameter, so the radius of the circle in question here is half of 10, and that number is 5. Thus, the area of the circle in this case is images, or images, which is formally written as images.

  22. E. images

    First, multiply images by images. To multiply terms with variables, multiply the coefficients and then multiply the variables. For any variable that is in both terms, write the variable with an exponent that is the sum of its exponents in the multiplied terms.

    images

    Like terms have either no variable or all the same variables, and every variable has the same exponent in each case. All such terms are like terms no matter what their coefficients are. Thus, any term that is a like term to imagescontains imagesand no other variables. The only way two terms can be combined into one term is if they are like terms. The only choice with the variable and exponent combination images is Choice (E).

  23. A. and B. –8.4 and (–1)997

    What is most relevant about the product of the factors in the question is that the product is negative. If 0 is not a factor, the product of factors in which an odd number are negative is a negative product. If an even number of factors are multiplied and 0 is not a factor, the product is positive. In (5)(–7)(2.3)(–17.4)(19)(–1), zero is not a factor, and 3 of the numbers are negative. Because 3 is an odd number, the product is negative. For a negative number to be multiplied by a number to get a positive product, the second number must also be negative. A negative number times a negative number is a positive number in all cases, and a negative number times a positive number is a negative number in all cases. That means any correct answer to this problem must be a negative real number, and that is the only requirement. Choice (A) is a negative number, so it qualifies as a correct answer. Choice (B) is also negative and therefore a correct answer. It is a negative number with an odd exponent. The odd exponent indicates that –1 is multiplied as a factor 997 times, an odd number of times. That makes the value negative.

  24. 1,402

    Since the question asks for a number of inches, convert the first two distances to numbers of inches.

    • images

    • images

    The last distance was 478 inches.

    The total distance is the sum of the three separate distances. You can use your calculator to add the three numbers of inches.

    images

    Ben threw the flying disc a total distance of 1,402 inches.

  25. A and C. The value of images is not equal to the value of images. The value of images is not equal to the value of images.

    If two triangles are similar, the ratio of a side measure of one triangle to the corresponding side measure for the other triangle is the same in all three cases, no matter which triangle’s side measures are in the numerators and which are in the denominators, as long as the numerator and denominator assignments are consistent. Also, a ratio of two side measures of one triangle is equal to the ratio of the measures of the corresponding sides of the other triangle, in all three cases.

    For this question, only Choices (A) and (C) have the correct corresponding sides to prove the triangles are similar.

  26. D. 27

    The sequence is geometric because each term is multiplied by 3 to get the next term. You could determine what the seventh and tenth terms are and then divide the tenth term by the seventh term to get the correct answer to the problem.

    The seventh term is 729 and the tenth term is 19,683.

    images

    There is another way to solve this problem: Since every term is multiplied by 3 to get the next term, the seventh term must be multiplied by 3 to get the eighth term, which must be multiplied by 3 to get the ninth term, which has to be multiplied by 3 to get the tenth term. That means the seventh term is multiplied by images to get the tenth term. The seventh term is multiplied by 3 with an exponent that is the number of places from seventh to tenth. Thus, the seventh term must be multiplied by images to reach a product that is the tenth term. Since images is 27, the correct answer is 27.

  27. B images.

    You must first determine the values of q and p by solving the system of linear equations. You can use the substitution or elimination method to do that. Most people would be more likely to prefer elimination because none of the coefficients in the system have a value of 1. If you multiply both sides of the second equation by 3, the two coefficients of p in the resulting system will be opposites. At that point, the equations can be added so that p is eliminated. You will be left with an equation with one variable, and an equation with one variable can be solved any time there is a solution.

    images

    Since the value of q is 6, you can put 6 in for q in any equation in this explanation.

    images

    The value of q is 6, and the value of p is –2. You can put each variable’s value into images to determine the rational expression’s value.

    images
  28. D. 510

    For the stem-and-leaf plot, the numbers on the left are the stems, and they represent hundreds. The numbers on the right are leaves, and they represent tens. That is indicated by the key at the bottom. The mode of a set of data is the number that is in the data in the greatest number of instances. It is the number that appears the most. Since no stem is ever repeated in a proper stem-and-leaf plot, you can look at just the leaves and find the number that appears the most on one level of the plot. For the stem-and-leaf plot in question here, that number is 1. It appears three times next to the stem of 5. That means 510 is the number that appears the most and is therefore the mode of the set of data.

  29. A. 6.05

    The volume of a right rectangular prism is equal to its base area times its height. Because any face of a right rectangular prism can be considered a base and the accompanying height will be the remaining side measure, the volume of a right rectangular prism is the product of its length, width, and height, and it is irrelevant which sides are considered which. Their product will be the same regardless. The volume of the prism on the left, which is a cube, is images, or images. If you multiply the two known side measures of the prism on the right, you get images. If you multiply that by x m, you will get a product of imagessince that prism has the same volume as the other one. The issue is therefore by what 1.76 must be multiplied to get 10.648.

    images

    The previously unknown side measure is 6.05 m, so the value of x is 6.05.

  30. C. Add $286

    Since Doris made the mistake of subtracting the price of the dress instead of adding it, she needs to add the price of the dress just to be where she was before the mistake. Then, she needs to add the price of the dress for the purpose of accounting for it. That means she must add $143 twice to fix the mistake and be back on track. That makes a total of $286 she must add to fix the mistake.

  31. A. Divide the resulting number by 4, subtract 7 from the result, and divide the difference by 3.

    In order to reverse the operation process, you need to do the opposite operation for every step, starting with the last and ending with the first. The last step of the described process was multiplying by 4. In order to undo that step, you must divide by 4. Then you will need to reverse adding 7 by subtracting 7. To undo multiplying the original number by 3, you need to divide by 3. At that point, you will have reached the original number.

  32. C. How much time do residents at a nursing home spend reading the newspaper each week?

    To answer Choice (C), one would have to record how much time residents spend reading the newspaper. Some days and weeks would have different amounts.

    Statistics is the area of math that deals with presentation and analysis of numerical data. Statistical questions are answered by collecting data with variability.

    The answer is not Choice (A) because it is a matter of totaling wins and comparing; there’s not much variation in the data. Choice (B) is simply yes or no, depending on the sample group of 16-year-old girls. Choice (D) simply requires giving a total number of years, and Choice (E) again only asks for a part of the total and is not considered statistical.

  33. A. images

    Since Bethany makes $8.25 per hour just for working, she makes $8.25 times the number of hours she works, just for working. The figure that represents that part of her pay is $8.25h. In addition to that, she gets $1.45 for every bouquet she sells. She sells b bouquets, so the part of her pay that comes from successfully selling bouquets of tulips is $1.45 times the number of bouquets she sells. The term which represents that part of her pay is thus $1.45b. Therefore, Bethany’s total pay for selling b bouquets for h hours is images.

  34. B. images and images

    The lower limit of what the ratio can be is the low limit of j divided by the high limit of k. That is images, or images. The upper limit of what the ratio can be is the high limit of j over the low limit of k. That is images, or images. Therefore, images must be above images and below images. It cannot possibly be anything outside of that range.

  35. A. 9.42 feet/second

    To determine the speed of the merry-go-round’s outer edge, you need to figure out the distance traveled by a point on the outer edge in one rotation. That distance is the same as the circumference of the merry-go-round. The formula for the circumference of a circle is images. In this case, images is images. Since the answer choices do not have images in them, you need to round images. That requires knowing its approximate value. It is an irrational number, so you cannot calculate with its exact value unless you use a calculator with a images key, which the current Praxis Core calculator does not have. If you round imagesto four decimal places, it is 3.1416. You can use that rounded number, and in many cases, you can use 3.14, depending how close to the exact answer the choices require you to get. For the sake of this calculation, we used 3.1415.

    images

    That is the approximate number of feet in the circumference of the merry-go-round. That number of feet divided by 4.87 seconds is the approximate number of feet per second the outer edge of the merry-go-round travels.

    images

    The choice that is closest to that rounded speed is Choice (A).

  36. E. 13

    Notice how in images, the x in images is replaced by –3. The x in images also needs to be replaced by –3.

    images
  37. E. E and F

    There was a 34.25 percent decrease from Day 3 to Day 4, so the profit level went from $94.28 to a number 34.25 percent less than itself. You can find 34.25 percent of $94.28 by converting 34.25 percent to a decimal, multiplying the result by $94.28, and subtracting that product from $94.28. The calculator for the Praxis Core exam math test follows the order of operations, so you can perform the entire calculation at once.

    images

    Matt’s profit level on Day 4 was $61.9891. That number is between 60 and 70, so it is represented on the number line between points E and F.

  38. A.
    image

    The bars of the histogram represent the percents of salespeople who sold each given number of boats. Since the histogram represents the sale numbers of 25 salespeople, each sale number is a portion of 25. For example, the number of salespeople who sold 3 boats is 4, so 3 appears in the table 4 times. That means 4 of the 25 salespeople sold that many boats. You can find the percent of salespeople who sold that many boats by dividing 4 by 25, moving the decimal point of the result two places to the right, and putting a percent sign (%) after the resulting number. An easier way is to simply multiply the number of salespeople for a given sale number by 4 because 25 times 4 is 100.

    images

    In other words, you can take any number of occurrences of a sale number (number of salespeople who sold that many) in the table and multiply it by 4 to get the percent of salespeople who achieved the sale number. You could also cross multiply and solve for x. This table illustrates the results:

    Number of Boats Sold

    Number of Salespeople

    Percent of Salespeople

    1

    5

    20

    2

    2

    8

    3

    4

    16

    4

    1

    4

    5

    6

    24

    6

    1

    4

    7

    1

    4

    8

    4

    16

    9

    1

    4

    The only histogram that correctly represents those results is Choice (A).

  39. B. images

    Since images and images, images, or images. That value of that rational expression over 2 can be calculated.

    images
  40. C. 979.2 m

    The area of the circular pond is images. The formula for the area of a circle is images. You can use that formula to find the radius of the circular pond by filling in the known values and solving for r.

    images

    The radius of the pond is 122.4 m. The diameter of a circle is twice its radius, so the diameter of the pond is imagesm, or 244.8 m. The diameter of the pond is also the measure of one side of the square path. The perimeter of a square is 4 times the measure of one of its sides, so the square path is images, or 979.2 m.

  41. B.
    image

    All of the graphs except Choice (B) represent certain x-coordinates that are paired with more than one y-coordinate each. That is what makes a relation not a function. Choice (B) does not represent any x-coordinate that is paired with more than one y-coordinate, so it represents a function. A graph that represents a function can pass the vertical line test, which is an analysis regarding whether a vertical line could pass through two points on the graph, anywhere, at once. If a vertical line cannot do that, it is because no value of x is paired with more than one value of y. Choice (B) passes the vertical line test, so it represents a function.

  42. E. images

    Robert has 5 cents for every nickel and 10 cents for every dime he has in his piggy bank. Since the total amount of money he has in his piggy bank for only nickels and dimes is $3.75, which is 375 cents, what he has in the piggy bank can be represented by the equation images. With that equation, you can solve for d to represent the number of dimes in the piggy bank.

    images
  43. A. images

    The answer choices are in the forms of solutions to inequalities. To determine the correct answer choice, solve the original inequality.

    images
  44. C and D. As Variable 1 increases, Variable 2 tends to decrease. As Variable 2 decreases, Variable 1 tends to increase.

    You can follow the numbers for Variable 1 in increasing order and see that the numbers for Variable 2 get lower over long enough stretches. However, Variable 2 does not decrease in every instance of a Variable 1 increase. Variable 2 merely tends to decrease as Variable 1 increases because it can take certain amounts of distance along the Variable 1 numbers to reach a decrease in Variable 2. Over long enough stretches, Variable 2 does decrease with increases in Variable 1.

    You can see that more clearly if you draw a line of best fit through the data points. A line of best fit is a line that exists inside the group of data points. Such a line in this case would go down from left to right, and it would thus indicate that as Variable 1 increases, Variable 2 tends to decrease.

  45. E. 36 in.
    image

    Based on the figure, the ribbon paper was divided into three parts, which together formed a right triangle. The part on the wall and the part on the metal formed the legs of the right triangle, and the strip connecting their ends formed the hypotenuse.

    You can use the Pythagorean theorem to determine the measure of the remaining side.

    images

    The three parts of the strip were thus 12 in., 9 in., and 15 in. The full length of the strip is the sum of those measures.

    images

    The strip of ribbon paper was 36 in.

  46. B. 2.3

    The dot plot shows that 3 people saw 1 bird, 4 people saw 2 birds, 2 people saw 3 birds, and 2 people saw 4 birds. This represents the data as

    1, 1, 1, 2, 2, 2, 2, 3, 3, 4, 4.

    To find the mean of this information, simply add and divide by the number of people.

    images
  47. B. The interquartile range of the represented set of data is between 45 and 50.

    The interquartile range is the distance from the first quartile to the third quartile. Those quartiles are marked by the edges of the box, and they are 30 and 70. The interquartile range is therefore 40 because images. Since 40 is not between 45 and 50, Choice (B) is a false statement, so it is the correct answer. The idea that Choice (B) is a true statement can result from thinking the mark inside the box is the interquartile range. That mark represents the median, which is approximately 49. Choice (E) is therefore a true statement. Choice (A) is true because the third quartile is 70 and most of the data in a set is less than the third quartile in all cases. Most of the data in any set is greater than the first quartile, so Choice (C) is true. Choice (D) is true because box-and-whisker plots display the beginnings and ends of sections of data but do not display means or even enough information to determine them.

  48. C. 83.24

    First, determine the value of g.

    images

    The value of g is 2.0549450549. If you multiply that number by 39.7, you will get the value of 39.7g.

    images

    The choice that is closest to that number is 83.24.

  49. 5.49

    The diagram shows the measures of the leg that the two triangles have in common and the hypotenuses of the two triangles. The measure of the other leg of the smaller triangle is not known, and neither is the other leg of the larger triangle. The value of p is the positive difference of the two missing leg measures. You can use the Pythagorean theorem to find both leg measures, and then you can find their difference to determine the value of p. You can use variables to represent the two unknown leg measures.

    • images

    • images

    The difference of images is the value of p. The value of p is therefore 5.4891… That number rounded to the nearest hundredth is 5.49.

  50. D. 1:45 p.m.

    The best way to approach this starts with determining how far apart the trains are at the moment the second train leaves the station. The second train leaves 2 hours and 15 minutes after the first one. That is images hours later. Since Train A, the first train to leave the station, is traveling 160 miles per hour, the distance it has traveled at 11:30 a.m. is images. You can determine that distance by converting images to decimal form and multiplying 160 by it.

    images

    At 11:30 a.m., Train A has already traveled 360 miles. The question asks what time the trains will be 1,215 miles apart. The trains have another images miles, or 855 miles to travel apart to reach that point. If you determine how long it will take for the trains to be another 855 miles apart, you can add that amount of time to 11:30 a.m. to find the answer to the question. From 11:30 a.m. onward, the trains will be traveling the same amount of time to reach 1,215 miles apart from each other. That amount of time is unknown, and you can use a variable to represent it. One speed times that amount of time plus the other speed times that amount of time equals 855.

    images

    When the trains are 1,215 miles apart, the time will be 2.25 hours after 11:30 a.m. You can convert 2.25 hours to 2 hours plus images minutes. That is 2 hours and 15 minutes. The time of day that is 2 hours and 15 minutes after 11:30 a.m. is 1:45 p.m.

  51. E. images

    Factor the numerator and denominator. From there, you can simplify the result by removing the common factor images.

    images
  52. A, C, and E. The perimeter of the garden. The length of the garden. The area of the garden.

    The question that is missing information essentially asks for the perimeter of the garden. The fence distance that is needed is the same as the garden perimeter. That makes Choice (A) a piece of information that could be used to answer the question. The length of the garden could be used to determine the garden perimeter because the width is given. The width and length of a rectangle are sufficient for determining the perimeter of a rectangle. Thus, Choice (C) is a correct answer. Choice (E) is correct because if the width and area of a rectangle are known, the length can be determined. Because the formula for the area of a rectangle is images, l can be determined with A and w. The height of the fence and the depth of the garden have nothing to do with how much distance the fence needs to cover in this situation, so Choices (B) and (D) are incorrect.

  53. D. images

    The formula for the area of a triangle is images, for which A is area, b is base, and h is height. The base measure can be any side measure, but the height must be the measure of a segment that comes from a vertex and is perpendicular to the side you call the base. Because the legs of a right triangle are perpendicular, their measures can be b and h. To find the measure of the unlabeled leg in the diagram, you can use the Pythagorean theorem. You can use a variable to represent the unknown leg measure.

    images

    Now that you have the other leg measure, you can use 5 and 12 as b and h and determine the area of the triangle.

    images

    The area of the triangle is images.

  54. A. images

    To determine the value, first raise what is in parentheses to the second power. To raise a power to a power, multiply the exponents. Then, multiply the result by what is before the parentheses. You can start by multiplying the coefficients. When multiplying a variable with an exponent by the same variable with an exponent, add the exponents.

    images
  55. D. He stretched in place for 1 minute. Then, he gradually built up speed to walk 3 miles per hour over the next minute, walked 3 miles per hour for the next 3 minutes, and gradually built up speed toward a 6-mile-per-hour jog over the next minute. He then jogged 6 miles per hour for 3 minutes. Then, he gradually slowed down toward a complete stop in the last minute and stood in place at the 10-minute mark.

    The graph indicates time horizontally and speed vertically. It indicates that Dr. Fitz had no speed for the first minute of the warm-up, so he could have been stretching in place during that time. Over the next minute, he increased his speed to what corresponds to 3 vertically, so he increased his speed to 3 miles per hour. He maintained that speed until the 5-minute mark. That is why the graph is horizontal until it reaches 5 minutes. The graph then moves up toward a speed of 6 miles per hour until the 6-minute mark, after which Dr. Fitz maintains 6 miles per hour until the 9-minute mark (3 minutes later). Maintaining that speed is what makes the graph horizontal for that interval. Over the next minute, the speed is reduced until it reaches 0 at 10 minutes.

  56. A. 316.128

    The table shows that there are 7.4 hiblems in 1 nochel and 4.8 nochels in one zerbert. That means there are images hiblems in 1 zerbert. There are 8.9 times that many hiblems in 8.9 zerberts, so the number of hiblems in 8.9 zerberts is images. The product of that multiplication is 316.128. There are thus 316.128 hiblems in 8.9 zerberts.

Answer Key

Part 1: Reading

  1. B
  2. A
  3. D
  4. D
  5. A
  6. C
  7. B
  8. E
  9. A
  10. C
  11. E
  12. C
  13. D
  14. D
  15. E
  16. D
  17. B
  18. C
  19. D
  20. C
  21. A
  22. C
  23. B
  24. C
  25. B
  26. B
  27. A
  28. E
  29. D
  30. D
  31. A
  32. E
  33. C
  34. B
  35. E
  36. B
  37. E
  38. E
  39. C
  40. A
  41. A and C
  42. 2008
  43. B
  44. E
  45. C
  46. E
  47. A
  48. C
  49. D
  50. D
  51. C
  52. A
  53. A
  54. C
  55. A
  56. D

Part 2: Writing

  1. C
  2. B
  3. A
  4. A
  5. B
  6. D
  7. B
  8. C
  9. A
  10. C
  11. D
  12. E
  13. C
  14. A
  15. B
  16. D
  17. B
  18. C
  19. C
  20. A
  21. A
  22. D
  23. E
  24. A
  25. C
  26. B
  27. D
  28. B
  29. C
  30. D
  31. A
  32. A
  33. C
  34. D
  35. C
  36. E
  37. E
  38. D
  39. A
  40. B

Part 3: Mathematics

  1. A
  2. B
  3. A
  4. C
  5. B
  6. D
  7. D and E
  8. D
  9. E
  10. B
  11. B
  12. C
  13. D
  14. B
  15. D
  16. C
  17. 4
  18. B
  19. C
  20. A
  21. C
  22. E
  23. A and B
  24. 1,402
  25. A and C
  26. D
  27. B
  28. D
  29. A
  30. C
  31. A
  32. C
  33. A
  34. B
  35. A
  36. E
  37. E
  38. A
  39. B
  40. C
  41. B
  42. E
  43. A
  44. C and D
  45. E
  46. B
  47. B
  48. C
  49. 5.49
  50. D
  51. E
  52. A, C, and E
  53. D
  54. A
  55. D
  56. A
..................Content has been hidden....................

You can't read the all page of ebook, please click here login for view all page.
Reset
18.222.117.109